How does one show that the ${\bf k}$-vector labeling a Bloch state is an arbitrary real vector

crystalsfloquet-theoryhamiltoniansolid-state-physicssymmetry

I'm frustrated that I can't understand something that must be simple and fundamental. I'd appreciate any answer to the question, but also any clarifications of how my presentation of the theorem/proof, or general quantum theory, is incorrect. Apologies for the long question; I could not figure out how to present the question completely without all of these details.

Bloch's Theorem states that, for a system with a periodic Hamiltonian — $H({\bf r} + {\bf R}) = H({\bf r})$ for all ${\bf R} \in A$, where $A$ is the set of all lattice symmetry translations, i.e., all integer linear combinations of the primitive lattice vectors $({\bf a_1}, {\bf a_2}, {\bf a_3})$ — any energy eigenstate satisfying $H | \psi \rangle = E | \psi \rangle$ "can be chosen so that associated with each $\psi$ is a wavevector ${\bf k}$ such that$^1$:"
$$
\psi({\bf r} + {\bf R}) = {\rm e}^{i {\bf k} \cdot {\bf R}} \, \psi( {\bf r})
$$

for each ${\bf R}\in A$. If we write:
$$
\psi({\bf r}) = {\rm e}^{i {\bf k} \cdot {\bf r}} u_{ {\bf k}}({\bf r})\,,
$$

then we see that
$$
\psi({\bf r} + {\bf R}) = {\rm e}^{i {\bf k} \cdot {\bf r}} {\rm e}^{i {\bf k} \cdot {\bf R}} \, u_{ {\bf k}}( {\bf r} + {\bf R})\, ,
$$

which implies that $u_{\bf k}({\bf r} + {\bf R}) = u_{\bf k}({\bf r})$ for all ${\bf R} \in A$. This is an alternative statement of the theorem: any energy eigenstate can be written as a plane wave times a function, labeled by the wavevector ${\bf k}$, that has the periodicity of the lattice.

Proofs of the theorem, e.g., from Ashcroft and Mermin or the wikipedia page, rely on choosing the wavevector to be of the form:
$$
{\bf k} = {\bf x} \cdot {\bf b} = x_1 {\bf b_1} + x_2 {\bf b_2} + x_3 {\bf b_3} \quad ({\bf x} \in \mathbb{R}^3)
$$

where the ${\bf b_i}$ are the primitive reciprocal lattice vectors satisfying ${\bf a_i} \cdot {\bf b_j} = 2 \pi \delta_{ij}$. The fact that the Hamiltonian commutes with each unitary translation operator $T_{\bf R}$ implies that energy eigenstates are also eigenstates of these translation operators. Letting
$$
{\bf R} = {\bf n} \cdot {\bf a} = n_1 {\bf a_1} + n_2 {\bf a_2} + n_3 {\bf a_3} \quad ({\bf n} \in \mathbb{Z}^3)
$$

we can write the eigenvalues of the translation operators as
$$
T_{\bf R} \, \psi({\bf r}) = \exp[i\, 2\pi \, (n_1 \tilde{x}_1 + n_2 \tilde{x}_2 + n_3 \tilde{x}_3)] \, \psi({\bf r})
$$

where the $\tilde{x}_i$ values specify the eigenvalues of the ${\bf a_i}$ primitive translations, i.e.,
$$ T_{\bf a_i} \, \psi({\bf r}) = \exp[i\, 2 \pi \, \tilde{x}_i] \, \psi({\bf r})\,.
$$

But, if we choose ${\bf k} = {\bf \tilde{x}} \cdot {\bf b}$, then we can write
$$
T_{\bf R} \, \psi({\bf r}) = \exp[i\, {\bf k} \cdot {\bf R}] \, \psi({\bf r})
$$

for all ${\bf R}\in A$. Therefore:
$$
\psi({\bf r} + {\bf R}) = T_{\bf R} \, \psi({\bf r}) = {\rm e}^{i\,{\bf k} \cdot {\bf R}} \, \psi({\bf r})
$$

for all ${\bf R} \in A$, showing that we can find a ${\bf k}$ such that the theorem is satisfied.

My confusion here is that it seems that the vector ${\bf k}$ is uniquely specified given an energy eigenstate $\psi$, i.e., it is given by the values $(\tilde{x}_1, \tilde{x}_2, \tilde{x}_3)$ that yield the eigenvalues of the primitive translations, ${\bf a_i}$, acting on that eigenstate $\psi$. That interpretation of the proof must be incorrect, however, because it implies a single ${\bf k}$ vector for each energy eigenstate. And, it is my undestanding, from the application of Bloch theory, is that the wavevector ${\bf k}$ should be thought of as a free parameter, an arbitrary real-valued 3-vector label (though its values can be restricted to the "first Brillion zone"). Indeed it is this continous freedom that gives the bands of energies. What am I missing here?

Possible solution: If I write the simultaneous energy and translation eigenstates as $| \psi_{\alpha, {\bf k}}\rangle$ then I must write the action of the translation operator as:
$$
T_{\bf R} \, |\psi_{\alpha, {\bf k}} \rangle = \exp[i\, 2\pi \, (n_1 \tilde{x}_{\alpha, {\bf k},1} + n_2 \tilde{x}_{\alpha, {\bf k},2} + n_3 \tilde{x}_{\alpha, {\bf k},3})] \, | \psi_{\alpha, {\bf k}} \rangle
$$

and then I choose ${\bf k} = {\bf \tilde{x}_{\alpha, {\bf k}}} \cdot {\bf b}$. This means that the $\tilde{x}_i$ values are specific to an eigenstate labeled by $(\alpha, {\bf k})$, not just by $\alpha$. Therefore we actually have complete freedom to choose ${\bf k} \in \mathbb{R}^3$. This argument feels a bit circular to me.

1 – From Ashcroft and Mermin, Chapter 8.

Best Answer

In what follows, operators have hats, un-hatted things are real or complex numbers

Suppose (in full generality, not necessarily assuming that we have a single particle state) that we have a Hamiltonian operator $\hat H$ that is periodic in the sense that $[\hat T_{\mathbf{R}}, \hat H] = 0$ for all $\mathbf{R} = n_1 {\bf a}_1 + n_2 {\bf a}_2 + n_3 {\bf a}_3$, where $\hat T_{\mathbf{R}} = \exp(i {\bf \hat p \cdot R })$ is the translation operator and $\underline{n} = (n_1,n_2,n_3) \in \mathbb{Z}^3$.

The game we are playing here is simply Hamiltonian diagonalisation. The facts we are using are

  1. Commuting, diagonalisable operators can be simultaneously diagonalised.
  2. Unitary operators are diagonalisable, with orthogonal eigenvectors and complex eigenvalues of norm 1.
  3. Hermitian opetators are diagonalisable, with orthogonal eigenvectors and real eigenvalues.
  4. For lattice translations, $\hat T_{\underline{n}} \equiv \hat T_{\mathbf{R}(\underline{n})} = \hat T_{\mathbf{a_1}}^{n_1} \hat T_{\mathbf{a_2}}^{n_2} \hat T_{\mathbf{a_2}}^{n_2}$

Using results (1-3), we find that there exists a spanning set for the Hilbert space $\{ | E, \lambda(\underline{n}) \rangle \}$, defined by $$\hat H | E, \lambda(\underline{n}) \rangle = E | E, \lambda(\underline{n}) \rangle\\ \hat T_{\underline{n}} | E, \lambda(\underline{n}) \rangle = \lambda(\underline{n}) | E, \lambda(\underline{n}) \rangle$$

I called this a spanning set because it is not yet clear whether these vectors are linearly independent. In fact, they are not: result (4) implies that $\lambda(\underline{n}) = \lambda(1,0,0)^{n_1} \lambda(0,1,0)^{n_2} \lambda(0,0,1)^{n_3}$, i.e. we can throw away all but three $\hat{T}_{\underline{n}}$ eigenvectors and span exactly the same space. In other words, we gain no additional information about a state from translations by more than one unit cell.

In this sense the only independent, commuting operators are

$$\hat H, \hat T_{\mathbf{a_1}}, \hat T_{\mathbf{a_2}}, \hat T_{\mathbf{a_3}}.$$

Unitarity allows us to rename $\lambda(1,0,0) = e^{ik_1'}, \lambda(0,1,0) = e^{ik_2'}, \lambda(0,0,1) = e^{ik_3'}$, or more concisely

$$ \lambda(\underline n) = \exp(i\underline k \cdot \underline n) $$

for $\underline{k}'$ chosen from $(\mathbb{R}/0\sim 2\pi)^3$. Reintroducing the units (via $\sum_i k_i' n_i = k_i \mathbf{b}_i \cdot \mathbf{R}_{\underline n} \equiv \mathbf{k} \cdot \mathbf{R}_{\underline n}$) proves the statement,

Given a Hamiltonian $\hat{H}$ that commutes with all integer translation operators, there exists a basis $\{ |E, \mathbf{k} \rangle \}$ for the Hilbert space with respect to which $\hat H$ is diagonal, and $\hat T_{\mathbf{R}_{\underline n}} |E, \mathbf{k} \rangle = e^{i \mathbf{k} \cdot \mathbf{R}_{\underline n}}|E, \mathbf{k} \rangle$.

The allowed values of $k$

Recall that the defining eigenvalues of a particular quantum state are

$$ E, e^{ik'_x},e^{ik'_y},e^{ik'_z} $$

It remains to see how the $k'_{x,y,z}$ are arranged. Firstly, note that the physical thing is $e^{ik_j}$ - because the complex exponential is periodic, the natural way to think of the $k$ values is as real numbers modulo $2\pi$ - $k$ and $k + 2\pi$ label precisely the same state.

Suppose that instead of being infinite, you put your lattice on a torus such that there are finitely many ($L$, say) unit cells in every direction. The translation operator becomes compactified due to $\hat{T}_{L\mathbf{a_j}} = \hat{1} \forall i=1,2,3$, and further, this implies that $\lambda(1,0,0)^L = 1$, i.e. $k'_i = \frac{2\pi n}{L}, n \in \mathbb{Z}$. Note that in 1D, this gives you exactly $L$ distinct eigenvalues modulo $2\pi$, and so in 3D, you end up with $L^3$ possible $\lambda(\underline n)$ eigenvalues / $k_{\underline n}$ (projective) eigenvalues. This argument established that the spectrum of the operator $\hat{T}_{\mathbf{a}_1}$ consists of $L$ evenly spaced complex phases on the unit circle.

Note the subtlety here - there are $L^3 + 1$ commuting operators ($\hat{T}_{\underline{n}}, \hat{H}$). If there is one local degree of freedom per unit cell (for a total of $N$ degrees of freedom), the system is actually overconstrained by the $k$ eigenvalues - this effectively makes $E$ a function of $k$. If there are two degrees of freedom per unit cell (e.g. graphene), then there are two bands, and $E$ is needed to distinguish them.

This counting argument suggests that it is helpful to express the full Hilbert space $\mathcal{H}$ as the product space $\otimes_{k \in BZ} \mathcal{H}_k$.

where the dimension of $\mathcal{H}_k$ is independent of the lattice size. As one takes $L \to \infty$, you find that $k$ can be any rational multiple of $2\pi$. Since the rationals are dense, one can argue for replacing the sum by an integral, but these technicalities are not really needed for solving most problems.